Mathcenter Forum  

Go Back   Mathcenter Forum > ค้นหาในห้อง
สมัครสมาชิก คู่มือการใช้ รายชื่อสมาชิก ปฏิทิน ค้นหา ข้อความวันนี้ ทำเครื่องหมายอ่านทุกห้องแล้ว

แสดงผลลัพธ์ตั้งแต่ 1 ถึง 25 จากทั้งหมด 40
ใช้เวลาค้นหา 0.01 วินาที.
ค้นหา: ข้อความของคุณ: JanFS
ห้อง: อสมการ 15 เมษายน 2009, 20:31
คำตอบ: 2
เปิดอ่าน: 1,879
ข้อความของคุณ JanFS
น่าสนใจนะครับ ผมลองคิดดูเล่นๆ...

น่าสนใจนะครับ
ผมลองคิดดูเล่นๆ อสมการต่อไปนี้ก็จริง (ก็ยังง่ายอยู่ล่ะครับ แต่ strong ขึ้นมา)
ให้ $a,b,c > 0$
$$\frac{b^{4}+3}{a+b}+\frac{c^{4}+3}{b+c}+\frac{a^{4}+3}{c+a} \geq...
ห้อง: เรขาคณิต 07 มกราคม 2009, 21:09
คำตอบ: 12
เปิดอ่าน: 4,508
ข้อความของคุณ JanFS
ขออนุญาตคุณ nongtum ครับ...

ขออนุญาตคุณ nongtum ครับ :kaka:
เส้นตรงสามเส้นนั้น ตัดกันที่จุดเดียวครับผม ไม่ใช่ collinear :happy:
ห้อง: เรขาคณิต 07 มกราคม 2009, 21:06
คำตอบ: 18
เปิดอ่าน: 5,810
ข้อความของคุณ JanFS
ขอโทษครับ คุณความเห็นที่ 12 และ 13...

ขอโทษครับ คุณความเห็นที่ 12 และ 13 คุณโพสโจทย์ที่มีการเผยแพร่ในเว็บไซต์อื่นโดยไม่บอกที่มา เหมาะสมแล้วหรือครับ
จริงๆ ผมก็ไม่ได้จะว่าอะไรนะครับ...
ห้อง: อสมการ 05 พฤศจิกายน 2008, 22:09
คำตอบ: 3
เปิดอ่าน: 2,129
ข้อความของคุณ JanFS
ให้ $P=18p$...

ให้ $P=18p$ อสมการที่ต้องการพิสูจน์จะกลายเป็น
$$8p^{3}+r^{3} \geq 9pr^{2}$$
แต่เราใช้ weighted AM-GM ได้ว่า
$$8p^{3}+r^{3} \geq 9p^{8/3}r^{1/3}$$
แต่สุดท้ายนี่สิครับ ผมไม่แน่ใจว่า $p \geq r$ รึเปล่า
ห้อง: อสมการ 17 กันยายน 2008, 20:57
คำตอบ: 7
เปิดอ่าน: 2,905
ข้อความของคุณ JanFS
น่าคิดดีนะครับ...

น่าคิดดีนะครับ :laugh:
ผมเคยเห็นแต่ข้อที่ง่ายกว่าก็คือ
ให้ $a,b,c>0$ ที่ $a^{2}+b^{2}+c^{2}=3$ จะได้ว่า
$$\sqrt{1+8a^2}+\sqrt{1+8b^2}+\sqrt{1+8c^2}\geq 3(a+b+c)$$
ห้อง: คอมบินาทอริก 07 กันยายน 2008, 22:14
คำตอบ: 4
เปิดอ่าน: 2,712
ข้อความของคุณ JanFS
ก็เหมือนของปีที่แล้วไงครับ ไม่ว่าจากเป็น สพฐ.หรือ...

ก็เหมือนของปีที่แล้วไงครับ ไม่ว่าจากเป็น สพฐ.หรือ ชลบุรี หรือ อะไร ก็มีเรขาข้อนึงทุกงานเลย - -
ห้อง: คอมบินาทอริก 07 กันยายน 2008, 18:10
คำตอบ: 4
เปิดอ่าน: 2,712
ข้อความของคุณ JanFS
ใช่ครับ แต่อันนี้ก็ถูกเพราะว่ามีคำว่า...

ใช่ครับ


แต่อันนี้ก็ถูกเพราะว่ามีคำว่า "ที่แตกต่างกัน" ซึ่งก็จะไบเจกชันพอดีกันกับ เงื่อนไขละครับ :)
ห้อง: คอมบินาทอริก 09 สิงหาคม 2008, 17:33
คำตอบ: 4
เปิดอ่าน: 2,416
ข้อความของคุณ JanFS
อ้าว ขอโทษครับ...

อ้าว ขอโทษครับ ผมเข้าใจว่า

โดยที่แต่ละสับเซตจะมีสมาชิกร่วมกันอย่างน้อย 1 ตัว หมายถึงว่า มีสมาชิกหนึ่งตัวที่อยู่ในทุกแต่ละสับเซต

ผมงงเองครับ ขออภัย "- -
ห้อง: เรขาคณิต 02 สิงหาคม 2008, 23:10
คำตอบ: 14
เปิดอ่าน: 10,417
ข้อความของคุณ JanFS
ผมก็รู้มาว่าปีกาทอรัสพิสูจน์ได้ 1 วิธี

ผมก็รู้มาว่าปีกาทอรัสพิสูจน์ได้ 1 วิธี
ห้อง: อสมการ 02 สิงหาคม 2008, 22:57
คำตอบ: 11
เปิดอ่าน: 4,259
ข้อความของคุณ JanFS
โทดครับ "- - ผมนอนตีหนึ่งมาห้าวันเลยเบลอไปหน่อย...

โทดครับ "- -
ผมนอนตีหนึ่งมาห้าวันเลยเบลอไปหน่อย 55+
ห้อง: ทฤษฎีจำนวน 01 สิงหาคม 2008, 21:23
คำตอบ: 15
เปิดอ่าน: 4,412
ข้อความของคุณ JanFS
"- - ทำผมซีดไปเลยครับ ก่อนจะอ่านล่างสุดเนี่ย...

"- - ทำผมซีดไปเลยครับ ก่อนจะอ่านล่างสุดเนี่ย -*-
ห้อง: อสมการ 01 สิงหาคม 2008, 21:19
คำตอบ: 11
เปิดอ่าน: 4,259
ข้อความของคุณ JanFS
กฏหรอครับ ......

กฏหรอครับ ... ผมไม่ชอบคำนี้เลยจริงๆ
ถ้ายังมีกฏก็ไม่มีทางเก่งคณิตศาสตร์หรอกครับ

หลักการของอสมการก็คือ $x^{2}\geq0$ ทุก $x \in \mathbb{R}$

เช่น $a^{2}+b^{2}\geq2ab$ ทุกจำนวนจริง $a,b$ เป็นต้น
ห้อง: ทฤษฎีจำนวน 30 กรกฎาคม 2008, 18:00
คำตอบ: 23
เปิดอ่าน: 9,670
ข้อความของคุณ JanFS
แบบพื้นฐานซักสองข้อ (1) จงแสดงว่ามี $n \in...

แบบพื้นฐานซักสองข้อ

(1)
จงแสดงว่ามี $n \in \mathbb{N}$ ซึ่ง มีจำนวนประกอบ $k$ ที่
$k \equiv 1 \pmod{2} $
$k \equiv 3 \pmod{4} $
$k \equiv 15 \pmod{16} $
$k \equiv 255 \pmod{256} $
$...$
$k \equiv...
ห้อง: อสมการ 30 กรกฎาคม 2008, 17:48
คำตอบ: 5
เปิดอ่าน: 3,204
ข้อความของคุณ JanFS
ไอเดียต่อไปนี้อาจช่วยได้...

ไอเดียต่อไปนี้อาจช่วยได้ :
$f\left(t\right)=\left(1+t\right)^\frac{1}{t}$ เป็นฟังก์ชันลดที่ $t \in \mathbb{R}^{+}$
ห้อง: พีชคณิต 30 กรกฎาคม 2008, 17:44
คำตอบ: 2
เปิดอ่าน: 11,179
ข้อความของคุณ JanFS
จากเอกลักษณ์มุมสามเท่าจะได้ว่า $\sin{3A}=3\sin{A}-...

จากเอกลักษณ์มุมสามเท่าจะได้ว่า
$\sin{3A}=3\sin{A}-4\sin^{3}{A}$
$=\sin{A}\cdot\left(1+2\cdot\left(1-2\sin^{2}{A}\right)\right)$
$=\sin{A}\cdot\left(1+2\cdot\cos{2A}\right)$ ตามต้องการ :kaka:
ห้อง: พีชคณิต 30 กรกฎาคม 2008, 17:31
คำตอบ: 9
เปิดอ่าน: 63,356
ข้อความของคุณ JanFS
ตอบ $a \in \left(0,\pi...

ตอบ $a \in \left(0,\pi \right)$

พิสูจน์
$\Leftarrow $ เห็นได้ชัดว่า มุมในสามเหลี่ยมต้องมีค่า $a \in \left(0,\pi \right)$
$\Rightarrow $ ให้ $a \in \left(0,\pi \right)$ ใดใด
สร้างสามเหลี่ยมที่ $AB =...
ห้อง: อสมการ 27 กรกฎาคม 2008, 20:03
คำตอบ: 9
เปิดอ่าน: 4,858
ข้อความของคุณ JanFS
ก็ยังผิดครับ "- - Nesbitt ครับ

ก็ยังผิดครับ "- -
Nesbitt ครับ
ห้อง: คอมบินาทอริก 27 กรกฎาคม 2008, 14:18
คำตอบ: 4
เปิดอ่าน: 2,416
ข้อความของคุณ JanFS
พิสูจน์ อันดับแรกจะแสดงว่า ไม่มีทางที่จะสร้างเซต...

พิสูจน์
อันดับแรกจะแสดงว่า ไม่มีทางที่จะสร้างเซต $F$ ที่สมาชิกเกิน $\binom{n-1}{m-1}$
สมมติว่ามีเซต $F$ ที่สมาชิกเกิน $\binom{n-1}{m-1}$
เนื่องจากทุกเซตใน $F$ ต้องมีสมาชิกร่วมกันอย่างน้อย 1 ตัว...
ห้อง: คอมบินาทอริก 27 กรกฎาคม 2008, 14:05
คำตอบ: 4
เปิดอ่าน: 2,416
ข้อความของคุณ JanFS
ถ้าจำไม่ผิดข้อนี้ตอบ $\binom{n-1}{m-1}$ นะครับ...

ถ้าจำไม่ผิดข้อนี้ตอบ $\binom{n-1}{m-1}$ นะครับ เดี๋ยวจะเอาพิสูจน์มาให้ครับ
ห้อง: อสมการ 27 กรกฎาคม 2008, 14:02
คำตอบ: 6
เปิดอ่าน: 3,069
ข้อความของคุณ JanFS
555+ คุณ murderer@IPST...

555+ คุณ murderer@IPST ได้เหรียญทองมาแล้วคึกจังนะครับ ^^
ห้อง: ข่าวคราวแวดวง ม.ปลาย 20 กรกฎาคม 2008, 20:40
คำตอบ: 5
เปิดอ่าน: 4,715
ข้อความของคุณ JanFS
ผลการแข่งขัน IMO 2008 (เย้!เด็กไทยเทพ)

ประกาศผล IMO2008 แล้วนะครับ :laugh:
ปีนี้ไทยได้ 2 เหรียญทอง 3 เหรียญเงิน และ 1 เหรียญทองแดง :great:
ยอดเยี่ยมมมมมมมมมมมมมมม
วู้ววววววววววววววววววววววววววววววว ดีใจมาก :D
ห้อง: อสมการ 13 กรกฎาคม 2008, 01:16
คำตอบ: 4
เปิดอ่าน: 1,966
ข้อความของคุณ JanFS
Solution

ในเมื่อผ่านไปสองสัปดาห์แล้ว ยังไม่มีใครทำ
ดังนั้นผมขออนุญาตเฉลยเลยนะครับ

จาก $a,b,c>0$ และ
จาก $a^{3}+b^{3}+c^{3}=1$ ดังนั้น $a^{3}<1, b^{3}<1, c^{3}<1$
เราเคลมว่า...
ห้อง: เรขาคณิต 11 กรกฎาคม 2008, 23:10
คำตอบ: 18
เปิดอ่าน: 6,162
ข้อความของคุณ JanFS
อันที่จริงผมว่า เจ้าของกระทู้น่าจะลอกมาผิดนะครับ...

อันที่จริงผมว่า เจ้าของกระทู้น่าจะลอกมาผิดนะครับ :huh: (ถ้าไม่ตรงก็ขออภัยด้วย เป็นความคิดเห็นส่วนตัว)
เพราะโจทย์ไม่น่าจะให้คำตอบเป็นแบบนี้
ถ้าตอบแบบนี้น่าจะให้เป็น $a,b,c$ มาเลยมากกว่าที่จะเป็นตัวเลข...
ห้อง: อสมการ 10 กรกฎาคม 2008, 23:55
คำตอบ: 3
เปิดอ่าน: 1,833
ข้อความของคุณ JanFS
แต่ใจจริง พอผมเห็นแล้วไม่ค่อยชอบเลยครับ เพราะมัน...

แต่ใจจริง พอผมเห็นแล้วไม่ค่อยชอบเลยครับ
เพราะมัน symmetry เอาไปใช้ยาก :sweat:
ถ้าผมค้นพบอะไรที่ใช้ง่ายกว่าก็จะเอามาบอกทีหลังละกันครับ :happy:
ห้อง: ปัญหาคณิตศาสตร์ ม.ปลาย 06 กรกฎาคม 2008, 22:03
คำตอบ: 1
เปิดอ่าน: 1,502
ข้อความของคุณ JanFS
ข้อนี้เป็นการประยุกต์ของสูตร $$\tan 2A...

ข้อนี้เป็นการประยุกต์ของสูตร
$$\tan 2A =\frac{2\tan A}{1-\tan^{2}A}$$
ซึ่งจะได้ว่า $1-\tan^{2}A=\frac{2\tan A}{\tan 2A}$ นั่นเอง
จับยัดเข้าไป ก็ง่ายละครับ :kaka:
ซึ่งจะได้ว่าทั้งก้อนนั้นเท่ากับ...
แสดงผลลัพธ์ตั้งแต่ 1 ถึง 25 จากทั้งหมด 40

 
ทางลัดสู่ห้อง

เวลาที่แสดงทั้งหมด เป็นเวลาที่ประเทศไทย (GMT +7) ขณะนี้เป็นเวลา 05:45


Powered by vBulletin® Copyright ©2000 - 2024, Jelsoft Enterprises Ltd.
Modified by Jetsada Karnpracha